با ادامه کار این ماراتن موافقید؟


  • مجموع رای دهندگان
    331

diba1993

New Member
ارسال ها
51
لایک ها
2
امتیاز
0
#41
rezashiri گفت
mrbayat گفت
9-ب)(یه لم جالب و مفید) اگه داشته باشیم
آنگاه بیشترین توان
که
را عاد می کند برابر است با:
[center:1657a216dc]

حالا اگه قرار بدیم
واضح است که
حالا برای بزرگترین توان
که
را عاد می کند داریم:
پس حداکثر توان
که
را عاد می کند برابر
است و نمی تواند برابر
باشد.​
[/center:1657a216dc]
لم جالبی بود.

ولی خیلی راحت می شد با شماردن تعداد عامل ها حلش کرد...

لطفا با روشي كه گفتي حل كن
[b گفت
mrbayat[/b]
]

داریم
و اگر یه
بگیریم و بزرگتر مساوی صفر قرار بدیم به دست میاد
یعنی :
با چک کردن این سه مقدار جواب ها را بدست می آوریم:


راه حل خوبه یا باید بیشتر از بخش پذیری استفاده کرد؟؟
فكر كنم دلتا رو غلط گرفتي .


rezashiri گفت
[center:1657a216dc]
[/center:1657a216dc]تمام اعداد صحیح x,y را بیابید که داشته باشیم: (آلمان - 1998)

[center:1657a216dc]
[/center:1657a216dc]
طرفين معادله رو با 3 جمع ميكنيم:

كه نتيجه ميده:

اگه بخش پذيري حل كنيم :

با جايگذاري x جواب ها بدست ميان.
 

rezashiri

Well-Known Member
ارسال ها
1,458
لایک ها
325
امتیاز
83
#42
mrbayat گفت
داریم
و اگر یه
بگیریم و بزرگتر مساوی صفر قرار بدیم به دست میاد
یعنی :
با چک کردن این سه مقدار جواب ها را بدست می آوریم:


راه حل خوبه یا باید بیشتر از بخش پذیری استفاده کرد؟؟
اولا چرا
؟!؟

دوما میشه راه حلتو کامل بذاری!؟!

سوما چطوری 1,1 توی معادله صدق می کنه!؟
 

diba1993

New Member
ارسال ها
51
لایک ها
2
امتیاز
0
#43
rezashiri گفت
mrbayat گفت
داریم
و اگر یه
بگیریم و بزرگتر مساوی صفر قرار بدیم به دست میاد
یعنی :
با چک کردن این سه مقدار جواب ها را بدست می آوریم:


راه حل خوبه یا باید بیشتر از بخش پذیری استفاده کرد؟؟
اولا چرا
؟!؟

دوما میشه راه حلتو کامل بذاری!؟!

سوما چطوری 1,1 توی معادله صدق می کنه!؟
طرفين معادله به پيمانه x همنهشتند
 

rezashiri

Well-Known Member
ارسال ها
1,458
لایک ها
325
امتیاز
83
#44
rezashiri گفت
[center:4f974a2b64]

[/center:4f974a2b64]تمام اعداد صحیح x,y را بیابید که داشته باشیم: (آلمان - 1998)

[center:4f974a2b64]
[/center:4f974a2b64]
راه حل من اینه:

داریم:

چون x,y صحیح اند داریم:



[center:4f974a2b64]
[/center:4f974a2b64]
 

rezashiri

Well-Known Member
ارسال ها
1,458
لایک ها
325
امتیاز
83
#45
قسمت الف سوال 9 هنوز حل نشده...

راهنمایی: اگر بر 11 بخشپذیر باشد .....(چه راهنمایی خوبی
)
 

diba1993

New Member
ارسال ها
51
لایک ها
2
امتیاز
0
#46
rezashiri گفت
mrbayat گفت
9-ب)(یه لم جالب و مفید) اگه داشته باشیم
آنگاه بیشترین توان
که
را عاد می کند برابر است با:
[center:1fa6e1ab8b]

حالا اگه قرار بدیم
واضح است که
حالا برای بزرگترین توان
که
را عاد می کند داریم:
پس حداکثر توان
که
را عاد می کند برابر
است و نمی تواند برابر
باشد.​
[/center:1fa6e1ab8b]
لم جالبی بود.

ولی خیلی راحت می شد با شماردن تعداد عامل ها حلش کرد...
لطفا اي سوال با شمارش عامل ها حل كن.

mrbayat گفت
9-ب)(یه لم جالب و مفید) اگه داشته باشیم
آنگاه بیشترین توان
که
را عاد می کند برابر است با:
[center:1fa6e1ab8b]

حالا اگه قرار بدیم
واضح است که
حالا برای بزرگترین توان
که
را عاد می کند داریم:
پس حداکثر توان
که
را عاد می کند برابر
است و نمی تواند برابر
باشد.​
[/center:1fa6e1ab8b]
اگه ميشه شما هم اثبات لم و بذار

rezashiri گفت
قسمت الف سوال 9 هنوز حل نشده...

راهنمایی: اگر بر 11 بخشپذیر باشد .....(چه راهنمایی خوبی
)
,

حال اگر

با جايگذاري
در عبارت فوق نتيجه ميشود كه

حال چون
,
پس:


 

Aref

New Member
ارسال ها
1,262
لایک ها
1,008
امتیاز
0
#47
[=dibaQUOTE]
لطفا اي سوال با شمارش عامل ها حل كن.
[/QUOTE]
واضحه:

واقعا نتونستی اینو ثابت کنی؟؟؟؟؟؟؟؟؟؟؟؟؟؟؟؟
 

Aref

New Member
ارسال ها
1,262
لایک ها
1,008
امتیاز
0
#48
شرط تساوی هم رخ نمیده چون n حتما از یه توان دویی کوچیکتره
 

rezashiri

Well-Known Member
ارسال ها
1,458
لایک ها
325
امتیاز
83
#49
[center:ee286ce018]
[/center:ee286ce018]

همه ی جواب های معادله زیر را در اعداد صحیح بیابید:(مرحله سوم ایران-1382)

[center:ee286ce018]
[/center:ee286ce018]
 

diba1993

New Member
ارسال ها
51
لایک ها
2
امتیاز
0
#50
Aref گفت
[=dibaQUOTE]
لطفا اي سوال با شمارش عامل ها حل كن.
واضحه:

واقعا نتونستی اینو ثابت کنی؟؟؟؟؟؟؟؟؟؟؟؟؟؟؟؟[/quote]
اگه ميتونستم ثابت كنم كه اثبات كنم نمي پرسيدم
 

diba1993

New Member
ارسال ها
51
لایک ها
2
امتیاز
0
#51
rezashiri گفت
[center:6fe3f1312f]
[/center:6fe3f1312f]

همه ی جواب های معادله زیر را در اعداد صحیح بیابید:(مرحله سوم ایران-1382)

[center:6fe3f1312f]
[/center:6fe3f1312f]
معادله فوق را به صورت زير باز نويسي ميكنيم:

كه نتيجه ميدهد:

با جايگذاري مقادير فوق در معادله اصلي داريم:

1)r=0

عبارت فوق با توجه به صحيح بودن
پاسخي ندارد.
با بررسي حالت هاي r=1و r=2 به نتيجه ي مشابهي خواهيم رسيد.
 

rezashiri

Well-Known Member
ارسال ها
1,458
لایک ها
325
امتیاز
83
#52
rezashiri گفت
[center:f920bd1e2a]
[/center:f920bd1e2a]

همه ی جواب های معادله زیر را در اعداد صحیح بیابید:(مرحله سوم ایران-1382)

[center:f920bd1e2a]
[/center:f920bd1e2a]
راه حل من :

داریم:



حال داریم:





ولی چون هیچ مربع کاملی به فرم
وجود ندارد پس چنین x ای وجود ندارد پس y هم وجود ندارد پس در کل معادله جواب ندارد.​
 

Aref

New Member
ارسال ها
1,262
لایک ها
1,008
امتیاز
0
#53
ببین آقای رضا شیری، عنوان تاپیکتو یه جوری انتخاب کردی آدم نمیتونه هیچ سوالی بزاره
حداقل میزاشتی ماراتن بخشپذیری متوسط
 

M_Sharifi

راهبر ریاضی
ارسال ها
1,981
لایک ها
801
امتیاز
0
#54
rezashiri گفت
[center:0f7f8c0225]
[/center:0f7f8c0225]

همه ی جواب های معادله زیر را در اعداد صحیح بیابید:(مرحله سوم ایران-1382)

[center:0f7f8c0225]
[/center:0f7f8c0225]
[center:0f7f8c0225]




ثابت کنید برای هر عدد طبیعی
، اعداد طبیعی
وجود دارند که
ولی
و
.​
[/center:0f7f8c0225]​
 

Aref

New Member
ارسال ها
1,262
لایک ها
1,008
امتیاز
0
#55

حالا کافیه دو خط دو توی x از 8 بیشتر بشه
 

rezashiri

Well-Known Member
ارسال ها
1,458
لایک ها
325
امتیاز
83
#56
Aref گفت
ببین آقای رضا شیری، عنوان تاپیکتو یه جوری انتخاب کردی آدم نمیتونه هیچ سوالی بزاره
حداقل میزاشتی ماراتن بخشپذیری متوسط
بذار یه چند روز بگذره تا سوالات مقدماتی تموم بشه بعد می ریم سراغ سوالات متوسط و عنوان تاپیکم می شه : بخشپذیری مقدماتی-متوسط
 

M_Sharifi

راهبر ریاضی
ارسال ها
1,981
لایک ها
801
امتیاز
0
#57
[center:1c2ec4523d]


ثابت کنید بی نهایت عدد طبیعی
وجود دارد که
.
[/center:1c2ec4523d]
 

rezashiri

Well-Known Member
ارسال ها
1,458
لایک ها
325
امتیاز
83
#58
M_Sharifi گفت
[center:bdc2f38f8a]


ثابت کنید بی نهایت عدد طبیعی
وجود دارد که
.
[/center:bdc2f38f8a]
میشه 4 تا از این بی نهایت تا رو نام ببرید...
 

Aref

New Member
ارسال ها
1,262
لایک ها
1,008
امتیاز
0
#59
1,2,6,...
 

rezashiri

Well-Known Member
ارسال ها
1,458
لایک ها
325
امتیاز
83
#60
بالا